Inscription / Connexion Nouveau Sujet
Niveau terminale
Partager :

Transformation de Toeplitz et théorème de Stolz-Cesàro

Posté par
Yona07
19-11-21 à 17:36

Bonjour!

Théorème de Toeplitz:

Soient (a_n)_{n\geq 1} une suite réelle convergente et (c_n)_{k\in\{1;2;...;n\}, n\geq 1} une famille de nombres réels telle que:

\begin{cases} (i) \lim_{n\rightarrow +\infty}c_{n,k}=0, \text{ pour tout } k\in N^* ;\\ (ii) \lim_{n\rightarrow +\infty}\sum_{k=1}^{n}{c_{n,k}}=1;\\ (iii) \text{ il existe C }>0: \text{pour tout n }\in N^*, \sum_{k=1}^{n}{|c_{n,k}|}\leq C. \end{cases}

1. Montrer que:

\lim_{n\rightarrow +\infty} (\sum_{k=1}^{n}{c_{n,k}a_k})=\lim_{n\rightarrow +\infty}a_n.

(Je n'arrive pas à la démontrer..)

2. Application 1 :

Soient (a_n)_{n\geq 1} et (b_n)_{n\geq 1} deux suites telles que:

\begin{cases} (i)'\;\; b_n>0, \text{ pour tout n }\in N^* \\ (ii)' \;\; \lim_{n\rightarrow +\infty}(b_1+b_2+...+b_n)=+\infty \\ (iii)' \lim_{n\rightarrow +\infty}a_n=l. \end{cases}

Montrer que:
\lim_{n\rightarrow +\infty}\frac{a_1b_1+a_2b_2+...+a_nb_n}{(b_1+b_2+...+b_n)}=l . (Fait. J'ai pris: c_{n,k}=\frac{b_k}{b_1+b_2+...+b_n} et j'ai appliqué le théorème de Toeplitz.)

3.Application 2: Théorème de Stolz-Cesàro.

Soient (x_n)_{n\geq 1} et  (y_n)_{n\geq 1} deux suites réelles telles que:

\begin{cases} (i)'' \;\; (y_n)_{n\geq 1} \text{ strictement croissante et }\lim_{n\rightarrow +\infty}y_n=+\infty \\ (ii)'' \;\; \lim_{n\rightarrow +\infty}\frac{x_n-x_{n-1}}{y_n-y_{n-1}}=1 \end{cases}

Montrer que: \lim_{n\rightarrow +\infty}\frac{x_n}{y_n}=l.

(Fait. J'ai pris: b_n= y_n-y_{n-1} et a_n=\frac{x_n-x_{n-1}}{ y_n-y_{n-1}} en relation avec l'application précédente. J'ai appliqué le même résultat. )

4. Application 3:

Soit f: R\rightarrow R une fonction continue. Montrer que:

(\int_{0}^{1}{f(x+n)dx})_{n\geq 1}\text{ convergente }\Rightarrow (\int_{0}^{1}{f(nx)dx})_{n\geq 1}\text{ convergente }

Posté par
Yona07
re : Transformation de Toeplitz et théorème de Stolz-Cesàro 19-11-21 à 17:36

Et merci d'avance!! ^^

Posté par
Foxdevil
re : Transformation de Toeplitz et théorème de Stolz-Cesàro 19-11-21 à 18:28

Bonsoir Yona07,

Pour la 1), connais-tu la preuve du théorème de Césaro? C'est en essence la même...

Posté par
Yona07
re : Transformation de Toeplitz et théorème de Stolz-Cesàro 19-11-21 à 19:04

Bonsoir Foxdevil!
Si, je la connais. Je vais essayer .

Posté par
Foxdevil
re : Transformation de Toeplitz et théorème de Stolz-Cesàro 19-11-21 à 19:46

Petite précision. Dans le 3 la condition (ii)'', la limite est bien un l au lieu d'un 1 non?

Posté par
Yona07
re : Transformation de Toeplitz et théorème de Stolz-Cesàro 19-11-21 à 20:11

Oui, vous avez raison. Désolée.

Posté par
Foxdevil
re : Transformation de Toeplitz et théorème de Stolz-Cesàro 19-11-21 à 20:15

Ok

Posté par
Yona07
re : Transformation de Toeplitz et théorème de Stolz-Cesàro 19-11-21 à 20:16

Yona07 @ 19-11-2021 à 17:36



3.Application 2: Théorème de Stolz-Cesàro.

Soient (x_n)_{n\geq 1} et  (y_n)_{n\geq 1} deux suites réelles telles que:

\begin{cases} (i)'' \;\; (y_n)_{n\geq 1} \text{ strictement croissante et }\lim_{n\rightarrow +\infty}y_n=+\infty \\ (ii)'' \;\; \lim_{n\rightarrow +\infty}\frac{x_n-x_{n-1}}{y_n-y_{n-1}}=l \end{cases}

Montrer que: \lim_{n\rightarrow +\infty}\frac{x_n}{y_n}=l.


Désolée encore une fois ^^''.

Posté par
Yona07
re : Transformation de Toeplitz et théorème de Stolz-Cesàro 19-11-21 à 20:28

Je n'arrive pas à démontrer 1 ^_^ ''

La démonstration de Césaro est faite par définition de limite.

J'ai:
\lim_{n\rightarrow +\infty }a_n=l

Donc par définition de limite:

>0, n1; nn1, l-\epsilon<a_n<l+\epsilon et donc:
nl-n\epsilon<\sum_{k=1}^{n}{a_k}<nl+n\epsilon
(ça ne me paraît pas correct du tout)

De plus: \lim_{n\rightarrow +\infty}\sum_{k=1}^{n}{c_{n,k}}=1, donc, par définition de limite:
>0, n2; nn2, 1-\epsilon<\lim_{n\rightarrow +\infty}\sum_{k=1}^{n}{c_{n,k}}<1+\epsilon

Alors:

(nl-n\epsilon)(1-\epsilon)<\lim_{n\rightarrow +\infty}\sum_{k=1}^{n}{c_{n,k}a_k}<(nl+n\epsilon)(1+\epsilon)

En vain....








Posté par
Foxdevil
re : Transformation de Toeplitz et théorème de Stolz-Cesàro 19-11-21 à 20:40

Je reprends la partie de Césaro qui va nous aider.

On découpe la somme et majore la somme  \frac{1}{n} \sum_{k=1}^n (a_k - l), de manière à avoir la majoration.

| \frac{1}{n} \sum_{k=1}^n (a_k - l) | \le \frac{1}{n} \sum_{k=1}^{N_0} | (a_k - l) | +  \frac{1}{n} \sum_{k=N_0+1}^{n} | (a_k - l) |

Et ce, pour un N_0 bien choisi. Le choix de N_0 permet de dire que le second morceau est plus petit que \frac{\varepsilon}{2}. Il y a également un rang à partir duquel le premier morceau est plus petit que \frac{\varepsilon}{2}.

Donc, si n est plus grand que le max des deux, alors on a bien prouvé que la moyenne de Césaro est epsilon-proche de l à partir de ce rang, soit que la moyenne de Césaro converge vers l.

Maintenant essaie de mimer cette preuve. A-t-on avis, où se cachent les c_{n,k}? ie qui ici (dans le Césaro classique) joue ce rôle...? Comment adapter du coup?

Posté par
phyelec78
re : Transformation de Toeplitz et théorème de Stolz-Cesàro 19-11-21 à 22:38

Bonjour,
pour le 1)  : voilà ce que je propose :

la suite  (a_n)_{n\geq 1}   est une suite réelle convergente, la suite transformée(b_n)_{n\geq 1} définit par b_n=\sum_{k=1}^n c_{n,k}a_k est aussi convergente car lim_{n-->+oo} b_n=lim_{n-->+oo} a_n

si tous les termes de suite  (a_n)_{n\geq 1} sont égaux d'après (ii) lim_{n-->+oo} b_n=a[/tex]

-à  partir de là c'est à valider par vous,je doute:
Ensuite, je pense qu'il faut considérer que  (a_n)_{n\geq 1} converge vers une limite l puisqu'elle est convergente ( je pense que ce serait pratique de prendre 0, mais je ne vois pas comment le justifier).

|b_n- l|=|\sum_{k=1}^n c_{n,k}a_k -l|  \leq |\sum_{k=1}^m |c_{m,k}||a_k |+ |\sum_{k=m}^n| c_{n,k}||a_k |+ |l|

\forall \epsilon > 0, \exists n_0 \in N \get    \forall n \in N ( n\geq n_0    | a_n -l | < \dfrac{\epsilon}{2C}

d'après i la suite {|a_{n,k}|} est bornée, d'après i, à partir d'un certain rang n2, dès que n \geq n_2
|\sum_{k=m}^n| c_{n,k}|  \leq \dfrac {\epsilon}2D avec D>0

si on prend m=n0, on a :

|b_n- l|  \leq |b_n|+| l|\leq | D\sum_{k=1}^{n_0} |c_{m,k}|+ \dfrac{\epsilon}2C\sum_{k=n_0}^n| c_{n,k}|+ |l|

soit |b_n|\leq | D\sum_{k=1}^{n_0} |c_{m,k}|+ \dfrac{\epsilon}2C\sum_{k=n_0}^n| c_{n,k}| \leq\dfrac{\epsilon}2 +\dfrac{\epsilon}2=\epsilon

là je n'ai plus l donc j'ai un doute , mais c'est peut-être une piste, un autre intervenant pourra peut-être vous proposer mieux.

Posté par
larrech
re : Transformation de Toeplitz et théorème de Stolz-Cesàro 20-11-21 à 09:20

Bonjour,

Citation :
( je pense que ce serait pratique de prendre 0, mais je ne vois pas comment le justifier).


Peut-être tout simplement en commençant par remplacer la suite (a_n) par la suite (v_n) dont le terme général est  v_n=a_n-l

Posté par
Foxdevil
re : Transformation de Toeplitz et théorème de Stolz-Cesàro 20-11-21 à 12:44

Bonjour phyelec78,

Ta preuve me semble vraiment confuse

Plusieurs problèmes...mais il faudrait vraiment tout éclaircir pour les traiter un à un.

Déjà le soucis principal est sur la majoration de bn ne peut être vraie, car l est quelconque.
Or, ta dernière majoration "prouve" que l=0.

Posté par
Foxdevil
re : Transformation de Toeplitz et théorème de Stolz-Cesàro 20-11-21 à 12:46

Je propose de laisser Yona07 chercher un peu avec les indications qui ont été données plus haut

Si elle le trouve pas, alors on mettra une solution...

Posté par
Yona07
re : Transformation de Toeplitz et théorème de Stolz-Cesàro 20-11-21 à 14:08

Bonjour tout le monde!
Merci pour vos interventions..
Honnêtement, j'ai passé toute la nuit à cuisiner qqch avec ces données mais rien n'en sort d'utile ( ^_^ )''....

Posté par
phyelec78
re : Transformation de Toeplitz et théorème de Stolz-Cesàro 20-11-21 à 15:17

@Foxdevil, vous avez raison ma dernière majoration "prouve" que l=0, donc c'est faux. Je voulais aboutir à |b_n -l | \leq \epsilon  et conclure, mais je ne ai pas abouti.

Posté par
Foxdevil
re : Transformation de Toeplitz et théorème de Stolz-Cesàro 20-11-21 à 16:26

J'éclaircis

Citation :
Maintenant essaie de mimer cette preuve. A-t-on avis, où se cachent les c_{n,k}? ie qui ici (dans le Césaro classique) joue ce rôle...? Comment adapter du coup?


A priori on ne les voit pas. Mais en y regardant de plus près, on voit que les c_{n,k} sont en fait ici les \frac{1}{n}. En les rentrant dans la somme, on a bien une expression du style \sum_{k=1}^{n}{c_{n,k}a_k}.
Et nos \frac{1}{n} vérifient bien les hypothèses que doivent vérifier les c_{n,k}.

Du coup, on va montrer que \sum_{k=1}^{n}{c_{n,k}a_k} - \sum_{k=1}^{n}{c_{n,k} l} converge vers 0; où on note l la limite de (a_n). Ce qui, en divisant par \sum_{k=1}^{n}{c_{n,k} , équivaut bien à dire que \sum_{k=1}^{n}{c_{n,k}a_k} converge vers l.

Soit \varepsilon > 0
| \sum_{k=1}^{n}{c_{n,k}a_k} - \sum_{k=1}^{n}{c_{n,k} l} | = | \sum_{k=1}^{n}{c_{n,k}(a_k - l)} | \le \sum_{k=1}^{n}{ |c_{n,k} | |a_k - l |}.

Mais comme (a_n) converge vers l, il existe un rang N_0, tel que pour toute valeur k entière au dessus de ce rang, on ait | a_k - l| \le \frac{\varepsilon}{2C} .

Du coup, si n \ge N_0+1, on a :

| \sum_{k=1}^{n}{c_{n,k}a_k} - \sum_{k=1}^{n}{c_{n,k} l} | \le \sum_{k=1}^{N_0}{ |c_{n,k} | |a_k - l |} + \sum_{k=N_0 +1}^{n}{ |c_{n,k} | |a_k - l |} \le \sum_{k=1}^{N_0}{ |c_{n,k} | |a_k - l |} + \sum_{k=N_0 +1}^{n}{ |c_{n,k} | \frac{\varepsilon}{2C}} \le \sum_{k=1}^{N_0}{ |c_{n,k} | |a_k - l |} + \frac{\varepsilon}{2}
car \sum_{k=N_0 +1}^{n}{ |c_{n,k} | \le \sum_{k=1}^{n}{ |c_{n,k} | \le C pour tout n.

Occupons-nous maintenant de \sum_{k=1}^{N_0}{ |c_{n,k} | |a_k - l |} . On sait que  \lim_{n\rightarrow +\infty}c_{n,k}=0. Donc \lim_{n\rightarrow +\infty} \sum_{k=1}^{N_0}{ |c_{n,k} | |a_k - l |}=0 (N_0 est fixé et ne dépend donc pas de n). Ce qui signifie qu'à partir d'un certain rang N_1,

\sum_{k=1}^{N_0}{ |c_{n,k} | |a_k - l |} \le \frac{\varepsilon}{2}.

On prend N_2 = N_0 + N_1 + 4586234621452 .

Et on a, pour tout n plus grand que N_2 :

| \sum_{k=1}^{n}{c_{n,k}a_k} - \sum_{k=1}^{n}{c_{n,k} l} | \le \sum_{k=1}^{N_0}{ |c_{n,k} | |a_k - l |} + \frac{\varepsilon}{2} \le \varepsilon.

Ce qui implique bien que \lim_{n\rightarrow +\infty} \sum_{k=1}^{n}{c_{n,k}a_k} = l = \lim_{n\rightarrow +\infty} a_n .

Posté par
Foxdevil
re : Transformation de Toeplitz et théorème de Stolz-Cesàro 20-11-21 à 16:29

Petite question: qu'en est-il du 4? Réussi? Pas réussi? Tu n'a rien précisé pour celui là...

Posté par
Yona07
re : Transformation de Toeplitz et théorème de Stolz-Cesàro 20-11-21 à 16:37

J'ai oublié de préciser: pas réussi..(^_^)''

Posté par
Foxdevil
re : Transformation de Toeplitz et théorème de Stolz-Cesàro 20-11-21 à 16:45

Applique SC pour x_n = \int_0^n f(x)dx et y_n = n

Posté par
Yona07
re : Transformation de Toeplitz et théorème de Stolz-Cesàro 20-11-21 à 16:51

Foxdevil @ 20-11-2021 à 16:26


A priori on ne les voit pas. Mais en y regardant de plus près, on voit que les c_{n,k} sont en fait ici les \frac{1}{n}. En les rentrant dans la somme, on a bien une expression du style \sum_{k=1}^{n}{c_{n,k}a_k}.


Je n'y ai pas du tout pensé! J'étais confuse puisque j'ai deux suites qui convergent (a_n) et (c_{n,k}). Des fois je commence le résonnement avec la limite de (a_n) et des fois avec celle de (c_{n,k}) mais aucune idée comment me débarrasser des \frac{1}{n} ..En tous cas, je n'ai abouti à rien.

Merci énormément! ^^

Posté par
Yona07
re : Transformation de Toeplitz et théorème de Stolz-Cesàro 20-11-21 à 17:28

Foxdevil @ 20-11-2021 à 16:45

Applique SC pour x_n = \int_0^n f(x)dx et y_n = n


Qu'est ce que SC?

Posté par
Foxdevil
re : Transformation de Toeplitz et théorème de Stolz-Cesàro 20-11-21 à 17:29

Le Théorème de Stolz-Cesàro

Posté par
Yona07
re : Transformation de Toeplitz et théorème de Stolz-Cesàro 20-11-21 à 18:19

ça marche ! Merci  infiniment Foxdevil ^^

Posté par
Foxdevil
re : Transformation de Toeplitz et théorème de Stolz-Cesàro 20-11-21 à 20:13

Je t'en prie

Posté par
Yona07
re : Transformation de Toeplitz et théorème de Stolz-Cesàro 20-11-21 à 21:00

Juste pour qu'il n'y ait pas de doute:

Prenons  x_n=\int_{0}^{n}{f(x)dx} et :

y_n=n :( (y_n) \text{ est strictement croissante et }\lim_{n\rightarrow +\infty} y_n =+\infty )\\

On a:

\frac{x_{n+1}-x_n}{y_{n+1}-y_n}= \frac{\int_{0}^{n+1}{f(x)dx}-\int_{0}^{n}{f(x)dx}}{n+1-n}=\int_{n}^{n+1}{f(x)dx}

--> Changement de variable:
Posons: u+n=x, ainsi:

\int_{n}^{n+1}{f(x)dx}=\int_{0}^{1}{f(u+n)du}

et donc:
\lim_{n\rightarrow +\infty}\frac{x_{n+1}-x_n}{y_{n+1}-y_n}=\lim_{n\rightarrow +\infty}\int_{0}^{1}{f(u+n)du}=l

Et d'après le théorème de Stolz-Cesàro:

\lim_{n\rightarrow +\infty}\frac{x_{n+1}}{y_{n+1}}=\lim_{n\rightarrow +\infty}\frac{\int_{0}^{n+1}{f(x)dx}}{n+1}=l

--> Changement de variable:
Posons (n+1)t=x, ainsi:

\int_{0}^{n+1}{f(x)\frac{dx}{n+1}}=\int_{0}^{1}{f(nt)dt}

Par la suite:

\lim_{n\rightarrow +\infty}\int_{0}^{1}{f(nt)dt}=l

Posté par
Foxdevil
re : Transformation de Toeplitz et théorème de Stolz-Cesàro 20-11-21 à 21:01

Posté par
Yona07
re : Transformation de Toeplitz et théorème de Stolz-Cesàro 20-11-21 à 21:05

Merci encore une fois!!

Posté par
phyelec78
re : Transformation de Toeplitz et théorème de Stolz-Cesàro 20-11-21 à 22:27

@Foxdevil, votre démonstration me convient et j'ai vu mon erreur, c'est quand j'ai écris |bn -I |

Posté par
Yona07
re : Transformation de Toeplitz et théorème de Stolz-Cesàro 21-11-21 à 12:58

Yona07 @ 20-11-2021 à 21:00

--> Changement de variable:

Posons (n+1)t=x, ainsi:

\int_{0}^{n+1}{f(x)\frac{dx}{n+1}}=\int_{0}^{1}{f(nt)dt}

Par la suite:

\lim_{n\rightarrow +\infty}\int_{0}^{1}{f(nt)dt}=l


Oops! J'ai détecté une erreur:

\int_{0}^{n+1}{f(x)\frac{dx}{n+1}}=\int_{0}^{1}{f((n+1)t)dt}
 \\

Posons: nv=(n+1)t, ainsi:

\int_{0}^{1}{f(nt)dt}=(1+\frac{1}{n})\int_{0}^{1+\frac{1}{n}}{f(nv)dv}\\ \;\;\;\;\;\;\;\;\;\;\;\;\;\;\;\; =\int_{0}^{1+\frac{1}{n}}{f(nv)dv}+\frac{1}{n}\int_{0}^{1+\frac{1}{n}}{f(nv)dv}

En fait:

\lim_{n\rightarrow +\infty}\int_{0}^{1}{f(nt)dt}= \lim_{n\rightarrow +\infty}\int_{0}^{1+\frac{1}{n}}{f(nv)dv}+\frac{1}{n}\int_{0}^{1+\frac{1}{n}}{f(nv)dv}=l

Et d'ailleurs:

\lim_{n\rightarrow +\infty}\frac{1}{n}=0 \text{ et }\lim_{n\rightarrow +\infty}\int_{0}^{1}{f(nt)dt}=l, \text{ donc: } \int_{0}^{1+\frac{1}{n}}{f(nv)dv}\neq \infty \\ \text{Par la suite:}\lim_{n\rightarrow +\infty} \frac{1}{n}\int_{0}^{1+\frac{1}{n}}{f(nv)dv}=0 \\ \text{Et donc: }\lim_{n\rightarrow +\infty}\int_{0}^{1+\frac{1}{n}}{f(nv)dv}=\lim_{n\rightarrow +\infty}\int_{0}^{1}{f(nv)dv}=l

C'est bon comme ça?

Posté par
Foxdevil
re : Transformation de Toeplitz et théorème de Stolz-Cesàro 21-11-21 à 13:09

Citation :
Oops! J'ai détecté une erreur:

\int_{0}^{n+1}{f(x)\frac{dx}{n+1}}=\int_{0}^{1}{f((n+1)t)dt}
 \\


Effectivement. J'ai noté ce détail mais ne l'ai pas signalé, car il est anodin.
Pas besoin de se prendre la tête en rechangeant de variable . Il suffit de dire que les limites de u_n et u_{n+1} sont les mêmes.

Posté par
Foxdevil
re : Transformation de Toeplitz et théorème de Stolz-Cesàro 21-11-21 à 13:17

Par contre, tes justifications de fins, bien que non nécessaires, ne me semblent pas correctes

Plusieurs choses ne sont pas claires...je reviens dessus plus tard

Posté par
Yona07
re : Transformation de Toeplitz et théorème de Stolz-Cesàro 21-11-21 à 13:27

C'est compris!
Je ne suis pas certaine de mes justifications également. Mais ça marche comme vous avez dit. Merci ^^!

Posté par
Foxdevil
re : Transformation de Toeplitz et théorème de Stolz-Cesàro 21-11-21 à 20:26

Citation :
Posons: nv=(n+1)t, ainsi:

\int_{0}^{1}{f(nt)dt}=(1+\frac{1}{n})\int_{0}^{1+\frac{1}{n}}{f(nv)dv}\\ \;\;\;\;\;\;\;\;\;\;\;\;\;\;\;\; =\int_{0}^{1+\frac{1}{n}}{f(nv)dv}+\frac{1}{n}\int_{0}^{1+\frac{1}{n}}{f(nv)dv}
Petite erreur. On a plutôt:

Citation :
Posons: nv=(n+1)t, ainsi:

\int_{0}^{1}{f((n+1)t)dt}=(1+\frac{1}{n})\int_{0}^{1+\frac{1}{n}}{f(nv)dv}\\ \;\;\;\;\;\;\;\;\;\;\;\;\;\;\;\; =\int_{0}^{1+\frac{1}{n}}{f(nv)dv}+\frac{1}{n}\int_{0}^{1+\frac{1}{n}}{f(nv)dv}


De même ici on a plutôt:

Citation :
En fait:

\lim_{n\rightarrow +\infty}\int_{0}^{1}{f((n+1)t)dt}= \lim_{n\rightarrow +\infty}\int_{0}^{1+\frac{1}{n}}{f(nv)dv}+\frac{1}{n}\int_{0}^{1+\frac{1}{n}}{f(nv)dv}=l


Enfin, les déductions ne permettent pas de savoir telles quelles, pourquoi on aurait les deux limites à la fin:

Citation :
\lim_{n\rightarrow +\infty}\frac{1}{n}=0 \text{ et }\lim_{n\rightarrow +\infty}\int_{0}^{1}{f((n+1)t)dt}=l, \text{ donc: } \int_{0}^{1+\frac{1}{n}}{f(nv)dv}\neq \infty \\ \text{Par la suite:}\lim_{n\rightarrow +\infty} \frac{1}{n}\int_{0}^{1+\frac{1}{n}}{f(nv)dv}=0 \\ \text{Et donc: }\lim_{n\rightarrow +\infty}\int_{0}^{1+\frac{1}{n}}{f(nv)dv}=\lim_{n\rightarrow +\infty}\int_{0}^{1}{f(nv)dv}=l

Posté par
Yona07
re : Transformation de Toeplitz et théorème de Stolz-Cesàro 21-11-21 à 21:57

Je me suis dite que puisque \lim_{n\rightarrow +\infty}\int_{0}^{1}{f((n+1)t)dt}=l

Et:

\lim_{n\rightarrow +\infty}\int_{0}^{1}{f((n+1)t)dt}=\lim_{n\rightarrow +\infty} \int_{0}^{1+\frac{1}{n}}{f(nv)dv}+\frac{1}{n}\int_{0}^{1+\frac{1}{n}}{f(nv)dv},

et on a de plus:

\lim_{n\rightarrow +\infty}\frac{1}{n}=0

Alors (par intuition) \int_{0}^{1+\frac{1}{n}}{f(nv)dv} ne tend pas vers l'infini..

C'est plutôt intuitif (cette intuition est déjà fausse) .. Et comme vous avez dit:  ça ne justifie rien..

Merci pour la correction! ^^

Posté par
Foxdevil
re : Transformation de Toeplitz et théorème de Stolz-Cesàro 21-11-21 à 22:09

Citation :
C'est plutôt intuitif (cette intuition est déjà fausse) .. Et comme vous avez dit:  ça ne justifie rien..
Oui ça paraît difficile à justifier.
Mais précisons que ce n'est ni nécessaire ni même suffisant.
Ce n'est pas nécessaire (de ne pas tendre vers l'infini), parce qu'on peut très bien avoir que ça tend vers l'infini, mais beaucoup moins vite que le 1/n, et donc que le produit tend vers 0.
Ce n'est pas suffisant, car ça peut ne pas tendre vers l'infini, et quand même osciller violemment (entre + et - l'infini).

Un truc sympa (mais pas nécessaire) serait que ça soit borné (et il se trouve que c'est le cas). Mais je vois vraiment pas pourquoi ça serait en général le cas (autrement qu'en se servant du résultat, qui du coup le prouve ^^)....je sais pas si je suis clair....

Posté par
Foxdevil
re : Transformation de Toeplitz et théorème de Stolz-Cesàro 21-11-21 à 22:13

Citation :
Ce n'est pas nécessaire (de ne pas tendre vers l'infini), parce qu'on peut très bien avoir que ça tend vers l'infini, mais beaucoup moins vite que le 1/n, et donc que le produit tend vers 0.
Heu non, je retire...en fait ça prouve bien que \lim_{n\rightarrow +\infty} \int_{0}^{1+\frac{1}{n}}{f(nv)dv} = l, car le facteur tend vers 1 (pas besoin de séprarer le 1 et le 1/n).

Par contre déduire de ça que \lim_{n\rightarrow +\infty} \int_{1}^{1+\frac{1}{n}}{f(nv)dv} = 0, ça paraît plus chaud...



Vous devez être membre accéder à ce service...

Pas encore inscrit ?

1 compte par personne, multi-compte interdit !

Ou identifiez-vous :


Rester sur la page

Inscription gratuite

Fiches en rapport

parmi 1675 fiches de maths

Désolé, votre version d'Internet Explorer est plus que périmée ! Merci de le mettre à jour ou de télécharger Firefox ou Google Chrome pour utiliser le site. Votre ordinateur vous remerciera !